Merge remote branch 'public/master'
[course.git] / latex / problems / Serway_and_Jewett_4 / problem22.04.tex
1 \begin{problem*}{22.4}
2 An electron is accelerated through $V = 2400\U{V}$ from rest and then
3 enters a uniform $B = 1.70\U{T}$ magnetic field.  What are \Part{a}
4 the maximum and \Part{b} the minimum values of the magnetic force this
5 charge can experience?
6 \end{problem*} % problem 22.4
7
8 \begin{solution}
9 First we compute the electron's velocity $v$ upon entering the field.
10 Conserving energy
11 \begin{align}
12  qV &= \frac{1}{2} m v^2 \\
13  v &= \sqrt{\frac{2qV}{m}}
14     = 29.0\U{Mm/s}
15 \end{align}
16
17 The magnetic force is given by $\vect{F} = q\vect{v}\times\vect{B}$,
18 so it is maximized when \vect{B} is perpendicular to \vect{v}, at
19 which point
20 \begin{equation}
21  F = qvB = \ans{7.90\U{pN}}
22 \end{equation}
23 The force is minimized then \vect{B} is parallel (or anti-parallel) to
24 \vect{v}, at which point $\ans{F = 0}$.
25 \end{solution}